DII 2018: Oliebollen - methods and appartuses

In the DII 2018, the client is a Bollebozen B.V. (BB) who is active in the food industry, both in the manufacture and design of processes and processing equipment and in the large-scale production of processed food. One of thei products is a Ducth speciality, called "Oliebollen", which are consumed at the turn of the year. They conwist of a ball of dough mixed with dried fruit, which is fried. One concern with all fried products is that they have high levels of acrylamide, a substance some studies have indicated is carcinogenic in high doses. ([001])
The client has a granted P patent that is in opposition appeal, 3 filed applications -3 EP and 1 PCT-, as well as the intention to file another PCT. A research institute offered the client a EP phase of a PCT application, which they will offer to one of the client's competitors if your client doesnot buy it. Some applications are deemed to be withdrawn because of lack of payment of fees. The PCT applications are still very early in the international phase. Some applications/patent claim priority, others donot (yet). Issues relate to genus-species with nozzle-conical nozzle-trumpet shaped nozzle, novelty of ranges and of amended (sub-)ranges, essential features, potential Art.54(3) effects, unclaimed subject-matter.
So far for the "usual" parts (even though the answer will be quite surprising). In addition to the above, the DII started with a challenging DI-like case: appeal in opposition, where the proprietor merely referred to first instance submissions than filing a complete statement of grounds, The opponent filed test results with the appeal to demonsytrate that also the patent as maintained in amended form had an insufficiency problem. Such highly legal topics have not really been part of the DII part anymore since the D-paper is a single 5-hour paper (with 30 minites additional time since last year).

Below we give our answer. We do not give a full analysis, but summarize the conclusions of all questions below. A full answer needs full discussion as to whether priority is valid or not, what all the prior art is, novelty, inventive step, provisional protection, full protection, etc.

The (outline of) our answer

1. What is our current position in the appeal proceedings and how should we proceed?
  • We filed the notice and paid the fee in time ([004]). 
  • We received the written decision only on 16/10, which is more than 10 days after the date stamped, 2/10+10d->12/10,
  • So we need to inform the EPO that we only received the written decision on 16/10/17
  • The time limit to file the grounds expired on 16/10/17 (actual date of notification) + 4m -> 16/10/17 + 4m -> 16/2/18 (Tue).
  • So the filing of the ground on 15/2/18 was in time
  • However, we merely referred to the submissions made in 1st instance
    • which does not not satisfy the requirement that the reasons for setting aside the decision need to be indicated and the extent to which it was to me amended (R.99(2)]; also Case Law Book (8th edition, IV-E, 2.6.4 a)
  • So our appeal will be rejected as inadmissible [R.101(1)], as we cannot remedy it within the expired 4m period to file the grounds anymore.
  • So we are not a full party but just a party as of right.
  • And as a result, the opponent benefits from reformation in peius. We can only defend our position but not improve it beyond the status of the decision, i.e. we can only defend the maintenance in amended form (just claim 2) or go to a narrower scope - G 9/92
  • The opponent appealed validly and admissible as his grounds were complete.
  • The opponent filed new evidence in the form of test results that show that also granted claim 2 lacks enablement (method using fruits A+B).
  • As the evidence could have been filed in 1st instance, we may request to hold the new evidence inadmissible – Art.12(4) RPBA
  • In particular, since fruit B is the feature of dependent claim 2 of the patent as granted the opponent should have expected it to be a fallback position.
  • If the new evidence is successfully found inadmissible, the board will maintain the patent on the same basis as the opposition division, that is, with a main claim which is a combination of claims 1 and 2 as granted.
  • However, the new evidence may be found admissible.
  • Then, as the original ground was also enablement, giving new arguments and filing new evidence (the test results) to further support that ground is allowed.
    • Further, as “the patent” was attacked, all claims were in the extent of opposition
  • As their test results are clear, claim 2 cannot be maintained.
  • We should file an auxiliary request to amend to a single claim 1+2+3
    • because we are just a party as of right, 
    • so that we cannot limit to 1+3 in view of the prohibition of reformation in peius  
  • Together with this request, we should file, or at least offer to file, evidence that the combination with A+B+C does work. 
  • This evidence is available (see par. [006])
  • Claim 1+2+3 should also be plausible on the basis of the description alone.
  • As this amendment is a reaction to the new evidence introduced by the opponent in its statement of the grounds, it is admissible.
  • Claim 1+2+3 is also novel and inventive, so it will be allowed
  • So, EP-BB1 will be maintained in amended form on the basis of granted claim 2 if the board does not admit the new evidence into the procedure.
  • Otherwise, EP-BB1 can be maintained in amended form on the basis of granted claim 3, dependent on 1, with the FFP  comprising fruit A, B and C. 

2. What is the current position as regards:
     2.1 – EP-BB2
    2.2 – PCT-BB2
     2.3 – EP-BB3 and the international application (PCT-BB3) we intend to file; 
and
     2.4 – EURO-PCT-FK1?

2.1 What is the current position as regards EP-BB2?
  • EP-BB2 is deemed to be withdrawn due to non-payment of the fees
  • The FP period to remedy this lapsed in July 2017 + 10d + 2m, so Sep/Oct 2017
  • We let it lapse intentional, so cannot use R
  • So EP-BB2 can no longer be revived
  • EP-BB2 was filed 3/2/17, 
  • so more than 12m ago,
  • so can also not serve as a priority application for a new filing
  • PCT-BB2 claimed priority from it
  • EP-BB2 is the BB’s first application for method of preparing FPP at 150-220 degr, pref 175-200
    • Note: It is not mentioned in [008] that the EP-BB2 (and PCT-Bb2) describes effect or advantages of the claims range. However, the paper tells us that test results have shown that in this temperature range has the effect of low acrylamide levels is achieved  irrespective of fruit type used in the ball. A candidate needs to deal with the question whether these effects are disclosed in the applications, or whether the effects are only known to the applicant so far.
  • EP-BB2 is the BB’s first application for apparatus with a mixer and a trumpet-shaped nozzle for forming balls of dough
  • EP-BB2 did not describe a crew conveyor

2.2 What is the current position as regards PCT-BB2?
  • PCT-BB2 was filed on 4/12/17, well within 12m from 3/2/17=f.d. EP-BB2
  • PCT-BB2 was filed by same applicant, BB
  • PCT-BB2 has same claim as EP-BB2, i.e.
  • PCT-BB2 claims method of preparing FPP at 150-220 degr, and describes pref 175-200
  • For this claim, priority from EP-BB2 is valid
  • EP-BB2 is published well before, so is full prior art, 
  • but does not mention any temperatures
  • So the method claim is novel over EP-BB1.
  • PCT-FK1 was published asap after 31/3/15 + 18m -> 30/11/16, so before the eff. date of the claim
  • PCT-FK1 discloses the method of preparing FPP at 155-250 degr
  • The end point 155 destroys the novelty of the claimed range 150-220
  • So the claim to the method of preparing FPP at 150-220 degr is not novel
  • So as it stands, the method claim of PCT-BB2 should not get granted.
  • PCT-BB2 is the BB’s first application for apparatus with a mixer, nozzle and a screw conveyor,
  • Wherein the screw conveyor is said to be an essential feature (“must”), so it must be part of any claim 
  • The screw conveyor is not in EP-BB2, so a claim having this feature has no valid prio
  • PCT-BB2, accidentally, also has a drawing of a trumpet nozzle – but it is not described. So doubtful whether you can derive from the drawing that it is trumpet-shaped. But even if you can, it must be combined with a screw conveyor, so is narrower than what you can get from EP--BB3. (see 3.3). 
  • PCT-BB2 currently has no claim to the apparatusSo as it stands, PCT-BB2 will (if not amended) not lead to a patent for the apparatus
2.3 What is the current position as regards EP-BB3 and the international application (PCT-BB3) we intend to file?
  • EP-BB3 is filed on 26/1/18
  • EP-BB3 is deemed to be withdrawn as period to pay filing and search fee expired 26-2-18
    • But can be remedied with FP – see 3.3
  • EP-BB3 does not claim priority
    • But we can still add a priority claim to EP-BB2 – see 3.3
    • But we can still add a priority claim to PCT-BB2 as far as that has new matter vs EP-BB2 – see 3.3
  • EP-BB3 does not describe any method
  • EP-BB3 does not mention a screw conveyor, as not needed for trumpet-shape
  • EP-BB3 claims apparatus with a mixer and a trumpet-shaped nozzle
    • Which was already described in EP-BB2, but no prio is claimed yet
  • As it stands, PCT-FK1 is full prior art, as published well before
  • But only shows a conical shape,  does not show the trumpet-shape, so claim is novel
  • If PCT-FK1 is only prior art, it would be inventive as it allows higher liquid content of the dough, further lowering the acrylamide levels ([009]), but as this is nowhere written in EP-BB3 (nor EP-BB2), it seems we cannot rely on this effect
    • It does however relate to this original effect of reducing acrylamide level, so it is allowed if that effect is mentioned in the application
    • Note: the paper nowhere says in [008] that EP-BB2 or PCT-BB2 has this effect…. It only mentions it as effect in EP-BB1 and in “what we realized” [009] or tests [006].
  • PCT-BB2 as far as is validly claims priority from EP-BB2 is Art.54(3) prior art if it enters EP (or, R.165 fulfilled)
  • PCT-BB2 already shows apparatus with a mixer and a trumpet-shaped nozzle in the figure
  • So, the claim is not novel if PCT-BB2 enters
  • Note:
    • This can be remedied by adding a priority claim from EP-BB2 to PCT-BB2
    • And, optionally, withdrawing PCT-BB2 before publication so it does not become a Art.54(3) in the EP-phase against Euro-PCT-BB3
2.4 What is the current position as regards EURO-PCT-FK1?
  • EURO-PCT-K is first and only appl from FK
  • Claim 1 claims an apparatus w mixer + nozzle
  • Description shows apparatus w mixer + conical nozzle + screw conveyor
  • Claim 2 claims method of preparing FPP containing D at 155-250 deg 
    • resulting in crispy, fluffy products
  • 20th century stuff is prior art [008] for claim 1
  • Already showed mixer + conical nozzle + screw conveyor,
  • So claim 1 is not novel
  • And cannot be amended to a novel claim
  • 20th century stuff is prior art [008] for claim 2, but shows no methods/ no D / no temperatures
  • EP-BB1 is prior art for claim 2
  • But does not show temperatures, so claim 2 novel
  • Claim 2 inventive as resulting in crispy, fluffy products [012]
  • Has entered, so can lead to patent for just claim 2:
    • method of preparing FPP containing D at 155-250 deg  
  • Which covers what we want – method of preparing FPP containing A, C and D at 160-210 deg and the products directly-obtained from that [Art. 64(2) EPC)]– so need to buy this!
  • And after buying, delete claim 1, to be ready to get a valid patent asap
  • Entered on 31/3/15 + 31m -> 31/10/17, 
  • so still within 10d+6m from R.161/162,  
  • so can amend immediately after buying it and having registered the transfer at the EPO, 
  • waive rest of R.161/162 period, and 
  • can expect a R.71(3) soon

3. How can we improve our position as regards our patent applications?

Main solution


3.1 – EP-BB2
  • dead, no action possible to revive.
3.2 – PCT-BB2
  •  “apparatus w mixer + nozzle + screw conveyor” not new
  • Method claims as it stands not new
  • The range 150-220, preferably 175-200, could be amended without violating Art.123(2) to 175-220, 175-200 or 200-220. 
  • For sub-range to be novel, the sub-range must be narrow (can be argued for 175-200), end points far removed (can be argued for 175-200, but not for the sub-ranges 175-220 and 200-220 as they have end point in common) and must be a purposive selection (new effect or improved effect). 
  • But PCT-BB2 does not seem to indicate any purpose to / effect of the originally claimed range, nor of working in the 175-200 subrange.
  • If the effect found in the tests as described in [006] is anyhow disclosed in PCT-BB2 (which is likely, but not stated in [008]), we can argue that is has an (improved) effect of reducing acrylamide levels which reduces carciogenic effects independent of what fruit is used.
    • So that purposive selection can be successfully argued, and the amended claim would be novel
    • And inventive because of it reduces carciogenic effects independent of what fruit is used.
    • We can submit the test results as evidence that the effect is really achieved, if needed.
    • We could then get a patent granted for a claim to method of preparing FFP by frying a ball of dough at a temperature in the range of 175-200C
  • If however no effects are disclosed in PCT-BB2
    • Purposive selection can not evidently argued,
    • So such amendment will in principle not render the claim novel.
    • We could however try to convince the examiner, with the test results as evidence, that  the purposive selection criterion is satisfied, hinted in the application by the words the “preferably” – as long as there is an effect in the application that is related to the new/improved effect achieved in the sub-range it should be admitted.
    • Note: the last reflects the real life situation: in real life one would always consider to give it a try before the EPO to amend the range based on a new effect that is not derivable from the application itself.
  • So, the method claim can or cannot be amended into an allowable claim, depending on whether purpoeive selectioin can be succesully argued.
  • Note: if you consider method not new, also not after amendment to sub-range, then good option to withdraw before publication, so a.s.a.p.
3.3 – EP-BB3 and the international application (PCT-BB3) we intend to file
  • EP-BB3: can continue to get “apparatus w mixer + trumpet nozzle”
  • Need to use FP for filing and search fee, pay 50% extra as FP fee, asap (have at least 2m still)
  • To overcome novelty problem w.r.t Euro-PCT-BBb2 is to get an earlier effective date:
  • add prio to EP-BB2
    • file prio declaration
    • before 3/2/17+16m -> 3/6/18 or 26/1/18 + 4m -> 26/5/18, so before 3/6/18
  • The, novel and inventive
  • So can get a mixer and a trumpet-shaped nozzle from EP-Bb3 
    • Note: not from PCT-BB2 based on drawings, as it “must” then also have the screw conveyor
  • Can do R.70(2) waiver, fast grant
  • PCT-BB3:
  • PCT-BB3 cannot validly claim prio from EP-BB3 for apparatus with a mixer and a trumpet-shaped nozzle, as EP-BB3 is not first application, but EP-BB2
  • So, PCT-BB2 will be Art.54(3) if it enters
  • EP-BB3 will not, as it is deemed to be withdrawn; but will be if revived 
  • So, as above, claim not novel over Euro-PCT-BB2 if that enters
    • But, here, adding a priority claim to EP-BB3 does not help (not first appl) and too late to add a priority claim to EP-BB2
  • So claim to apparatus with a mixer and a trumpet-shaped nozzle
  • Will not be novel in EP with claims as in EP-BB3 as EP-BB3 will be novelty-destroying Art.54(3)
  • Could file to get it outside EP, as only prior art is Art.54(3) only effective in EP

Alternative solution (a consistent advice if you concluded that PCT-BB2 cannot be amended to a subrange because of failure of the purposive selection requirement)
  • Withdraw PCT-BB2, EP-BB3 before publication
  • Do not file PCT-BB3
  • File a new PCT-BBX
    • With the complete text of EP-BB3
    • As the effect of the trumpet is not yet described in EP-BB3 nor in any application, also add the advantage of a trumpet nozzle over a conical nozzle: higher liquid content of the dough, further lowering acrylamide levels
    • Note: [008] introduces the trumpet nozzle in an application, but the application EP-BB2 does not mention its advantages. It advantages are only mentioned in [009].
  • Claim “apparatus w mixer + trumpet nozzle”
  • Only prior art is 20th century stuff of [008], PCT-FK1, EP-BB1, and Dutch newspaper article
    • But none show trumpet nozzle, so novel
    • And the effect makes it inventive
  • A method claim covering the whole range of ACD fruit at 160-210 will not be novel due to PCT-FK1 (D at 150-250) and/or the Dutch Newspaper article (ACD at  160-210)
    • Note: [006] describes advantages if a frying temperature below 220 degrees or if C is used as an ingredient, as found in tests. But neither of these advantages is mentioned in any application. The Newspaper article however prevents a new application for it to be novel.

4. Please give us all reasons why we should buy EURO-PCT-FK1.
  • EURO-PCT-FK1 will give protection for a method of preparing FFP containing D by frying dough at between 155-250C.
     
  • We want to have FTO to manufacture and sell processing equipment and do large-scale production of Oliebollen with fruits A, C and D, fried at 160-210C
     
  • EURO-PCT-FK1’s apparatus claim is not valid, so no reason directly related with that
    • but saves us some trouble to invalidate or be bothered by this invalid claim from others
  • If someone else gets EURO-PCT-FK1’s , he can stop us from using the method to manufacture FPP containing D at 155-250 deg, and we do not want the risk that they stop us with that claim from using the method for making Oliebollen with ACD at 160-210.
  • We need to have the patent to stop others from using the method to manufacture any FPP containing D at 155-250 deg, such as manufacturing FPP with ACD at 160-210.
     
  • If someone else gets EURO-PCT-FK1’s , he can stop us from selling/offer to sell the products directly obtained by the claimed method, in particular our Oliebollen comprising ABC made with the method at 160-210. We do not want others to prevent us from doing so.
  • We need to have the patent to stop others from selling any FPP containing D directly obtained from the claimed method at 155-250 deg, such as FPP with ACD directly obtained from the method at 160-210.
     
  • We do not want the risk that our interview / Dutch newspaper article is considered an infringing act on provisional protection – PCT-FK1 was published before, so if FK satisfied also the national requirements for NL such as claims translations, we may be have an issue - will check with Ducth attorney
     
  • We can amend EURO-PCT=FK1 to delete non-novel claim 1 (apparatus w mixer + nozzle)

Any comments, differing opinions, questions are welcome!

Please do not post your comments anonymously: it is more easy to communicate with a person having a name. Nicknames are fine, real names as well.

If you are looking for the DI discussion: it can be found here.
If you are looking for first and general impressionw about D 2018: it is here.

The D paper is available here in English, ici in French and hier in German.

Roel, Diane, Jessica, Sander, Jelle, Tanja

(c) DeltaPatents 2018

---------
For your reference:

GL (2016) G-VI – novelty of a  sub-range:
Part G – Patentability
Chapter VI – Novelty
8. Selection inventions
Selection inventions deal with the selection of individual elements, sub-sets, or sub-ranges, which have not been explicitly mentioned, within a larger known set or range.
         i. [..]
       ii.  A sub-range selected from a broader numerical range of the prior art is considered novel, if each of the following three criteria is satisfied (see T 198/84 and T 279/89):
a.   the selected sub-range is narrow compared to the known range;
b.   the selected sub-range is sufficiently far removed from any specific examples disclosed in the prior art and from the end-points of the known range;
c.   the selected range is not an arbitrary specimen of the prior art, i.e. not a mere embodiment of the prior art, but another invention (purposive selection, new technical teaching).
An effect occurring only in the claimed sub-range cannot in itself confer novelty on that sub-range. However, such a technical effect occurring in the selected sub-range, but not in the whole of the known range, can confirm that criterion (c) is met, i.e. that the invention is novel and not merely a specimen of the prior art. The meaning of "narrow" and "sufficiently far removed" has to be decided on a case-by-case basis. The new technical effect occurring within the selected range may also be the same effect as that attained with the broader known range, but to a greater extent.
     iii.   In the case of overlapping ranges (e.g. numerical ranges, chemical formulae) of claimed subject-matter and the prior art the same principles apply for the assessment of novelty as in other cases, e.g. selection inventions. It has to be decided which subject-matter has been made available to the public by a prior art disclosure and thus forms part of the state of the art. […]
As to overlapping ranges or numerical ranges of physical parameters, novelty is destroyed by an explicitly mentioned end-point of the known range, explicitly mentioned intermediate values or a specific example of the prior art in the overlap. It is not sufficient to exclude specific novelty destroying values known from the prior art range, it must also be considered whether the skilled person, in the light of the technical facts and taking into account the general knowledge in the field to be expected from him, would seriously contemplate applying the technical teaching of the prior art document in the range of overlap. If it can be fairly assumed that he would do so, it must be concluded that no novelty exists. In T 26/85, the skilled person could not seriously contemplate working in the area of overlap, since the prior art surprisingly contained a reasoned statement clearly dissuading him from choosing said range, although the latter was claimed in said prior art.
The criteria mentioned in (ii) above can be applied analogously for assessing the novelty of overlapping numerical ranges (see T 17/85). […]

GL (2016) H-IV, 2.4 – amending ranges: Art.123(2)
Part H – Amendments and Corrections
Chapter IV – Allowability of amendments – Art. 123(2) and (3)
2. Allowability of amendments under Art. 123(2)
2.4 Assessment of "added subject-matter" – examples
If an application relates to a rubber composition comprising several ingredients and the applicant seeks to introduce the information that a further ingredient may be added, then this amendment should normally be objected to as offending against Art. 123(2).
In the case of a disclosure of both a general and a preferred range, a combination of the preferred disclosed narrower range and one of the part-ranges lying within the disclosed overall range on either side of the narrower range may be derivable from the original disclosure of the application.
In an application which describes and claims an apparatus "mounted on resilient supports", without disclosing any particular kind of resilient support, objection should be raised if the applicant seeks to add the specific information that the supports are, or could be, e.g. helical springs.
If, however, the applicant were able to demonstrate that the drawings, as interpreted by the skilled person, show helical springs, the specific mention of helical springs would be allowable, at least in the context.




Comments

  1. I got from Q1 this:

    We filed the notice and paid the fee in time ([004]).
    We received the written decision only on 16/10, which is more than 10 days after the date stamped, 2/10+10d->12/10,
    So we need to inform the EPO that we only received the written decision on 16/10/17
    The time limit to file the grounds expired on 16/10/17 (actual date of notification) + 4m -> 16/10/17 + 4m -> 16/2/18 (Tue).
    So the filing of the ground on 15/2/18 was in time

    With time running out, I decided that the appeal von BB is ok and he can get claim 1 + 3


    Honestly, DI went pretty well, but I didn´t expect, that we will have such complex questions in DII ...

    ... well well Q2 - Q3 went a bit better but far from good ...

    Changing the format of DII without giving a hint is a bit unfair ...

    ReplyDelete
    Replies
    1. About your last sentence: There is no fixed format for DII. There have been Ds with big legal cases like here in the past.

      Delete
    2. I know, the last time was 2012 ... so I didn´t expect it. Lets see what comes out. At least I have B and C with enough marks to compensate.

      Nevertheless, for me it feels a bit unfair. D is difficult enough even in the "standard setting" of the last years

      Delete
  2. I think

    So our appeal will be rejected as inadmissible [R.101(1)], as we cannot remedy it within the expired 4m period to file the grounds anymore.

    Is a pure speculation and not based on any facts in the text. We usually are trained only take facts and no speculations.

    Case Law Book (8th edition, IV-E, 2.6.4 a) also states that in a small number of isolated decisions the boards have accepted a general reference to submissions at first instance as potentially constituting grounds for an admissible appeal.

    So even unlikely it could be that the appeal is accepted. There are no evidence in the text that a rejection took place ot that BB is worried about that point.



    ReplyDelete
    Replies
    1. Well there's no evidence because this is the conclusion you have to make, using your legal knowledge! If you found decision(s) supporting admissibility, you can say it, there may be extra points for doing that. :)

      Delete
    2. The (in)admissibility was a really difficult question to decide upon for me: I first thought its the only way to use the information of the case (regarding the mere reference to the opposition proceedings). However, I didnt find anything in the guidelines ... so I decided that the appeal is admissible

      Case law IV-E-2.6.6:


      In T 3/95 the board found that the fact that the points made in the statement of grounds of appeal did not go beyond those made before the opposition division did not detract from the admissibility of an appeal. A requirement that new arguments be submitted to render an appeal admissible would have implied that the appealed decision must have been correct. Nor did the appeal need to have a strong prospect of success.

      Delete
    3. @Anonymous 28 February 2018 at 18:30
      The Case Law Book (8th edition, IV-E, 2.6.4 a) clearly shows that merely referring to first instance submissions is inadmissibility. It also clearly says there is some exceptions, but they also said under which circumstances those were allows:

      "In a small number of isolated decisions, however, the boards have accepted a general reference to submissions at first instance as potentially constituting grounds for an admissible appeal (T 355/86, T 140/88, T 216/10), but these decisions were given in special cases in which those submissions already adequately addressed the grounds underlying the contested decision."
      There is no indication in the question that these was a similar case. So, it was not. Or, if you thnink it could be, you need to discuss both sides: what if it will be considered inadmissible -as most likely-, and what if considered admissible -as less likely-.

      Delete
    4. But than the following statement
      "So we need to inform the EPO that we only received the written decision on 16/10/17"
      is useless. Why should I advice my client to inform the EPO that the missed deadline was not really missed if I already know that I can not be successful with this information. If I inform the EPO I would generate costs for the client (I have to write a letter) that are not necessary and could be prevented.
      Do you believe that the EQE tries to kid us around with determining the deadline if nothing can be improved. Than I know where my time has been gone. And also it is not fair against somwone who did not see that by informing the EPO about late receiving and who said - for the missed deadline - there is no improvement possible.

      Delete
    5. There is such a need: difference between appeal not filed and appeal not admissible, so difference between appeal fee redunded or not.

      Note: in the exam, cost of attorney have -as far as I recall- never been a factor of consideration, only official fees.

      Delete
  3. Hi Roel and all, have you at delta patents discussed how you expect the marks to be divided in such a d2?

    ReplyDelete
    Replies
    1. i think 12 for first question, 28 for the second one, 16 for the third one and last 4 goes to last one
      Emrah

      Delete
    2. @Estela: no, we did not discuss that. Is difficult to predict: depends not only on what the exam committee initially planned, but they always do a pre-marking of many candidates to see whether the preferred solution and marking scheme needs to be adjusted. It also depends on what points of discussion they attribute to which queastion - e.g, "FK could stop us if we donot buy EURO-PCT-FK" is probably expected under Q.4, but could also be part of Q.2.4 - for the marks you get it doesnot matter, it only is relevant to which question they are allocated. The committee explained in Tutor's Meetings that that will e.g. be done if all candidates overlooked a problem - then, the marks for that are reduced and reallocate to other parts. So, the adjustment goes -as far as I understand what they explained- always to the advantage of (the majority of) the candidates.
      A distribution like the one that Emrah proposes may be a possible distribution.

      Delete
    3. I think 12 points for the first one would be really mean, especially given that the question hangs on how the candidates concluded on admissibility.

      Delete
  4. In regard of Q2 PCT-BB2
    is a new claim directed to the subrange 175 - 200°C patentable in light of the Guidlines G VI 8,
    "the selected range is not an arbitrary specimen of the prior art, i.e. not a mere embodiment of the prior art, but another invention (purposive selection, new technical teaching). "

    "The new technical effect occurring within the selected range may also be the same effect as that attained with the broader known range, but to a greater extent."

    But the DII description does not give an effect with a greater extend for the subrange. It is only stated this is a preferred subrange - whatever the reasons are (at least in the German paper).

    ReplyDelete
    Replies
    1. Hi Tom,

      For sure no, it will novel, exactly for the reasons you give: not a purposive selection as no effects at all mentioned, let alone an effect specifically for the amended narrower range.

      There is no way to amend the ranges to get novelty.

      Tom

      Delete
    2. @Tom and Tom: we had some discussion on this. You see the outcome reflected in our answer. It indeed seems that EP-BB2 and PCT-BB2 did not mention any effect as the paper does not tell it does (I didnot check all last papers, but I think it always did for every application and every subject-matter in it - would need to check to be sure). If so, one cannot succesfully argue purposive selection, nor inventive step, as that needs to relate to an effect described in the application or an effect related or derivable from that. However, it could be that it was intended that you dealt with the uncertainty of whether there are effects in it or not, and were expected to discuss the two situations: there are/are not effects disclosed - and how the EPO will deal with test results filed during examination to demonstrate effects that were/were not disclosed. It puzzled us... Part of the reason to include the discussin is that it is not very common to be expected to have quite extensive discussions (novcelty of sub-ranges) on things that in the end donot work out well - most marks go usually to discussing why and how you come to a solution that does work.
      Other points of view are welcome!

      Delete
    3. @Tom: you wrote: "[...] may also be the same effect as that attained with the broader known range, but to a greater extent." But the DII description does not give an effect with a greater extend for the subrange. It is only stated this is a preferred subrange - whatever the reasons are."

      The -only- basis would be the word "preferably", which is maybe enough to allow you to show with late-filed test results that indeed that range shows the effect to a greater extend. Provided the application did disclose the effect or an effect related to that - without an original effect, purposive selection and inventive step will fail I am afraid.

      Delete
  5. I would say it is patentable, there is a difference between an 'effect' and a 'technical effect' GL-GVI, 8, i.e.

    An effect occurring only in the claimed sub-range cannot in itself confer novelty on that sub-range. However, such a technical effect occurring in the selected sub-range, but not in the whole of the known range, can confirm that criterion (c) is met, i.e. that the invention is novel and not merely a specimen of the prior art.
    Lower levels of Acrylamide under 220C are not taking place in the whole know ragne 155-250C so it would be novel. The range has to be sufficintly apart from the boundary, and 20C should be.

    Pablo

    ReplyDelete
    Replies
    1. I could agree, but it is almost pettifoggery. Lower levels of Acrylamide is a new technical effect ... but the wording from the guidelines could be missleading ... especially for non-native speakers of the official EPO languages this could be very challenging

      Delete
    2. @Pable:
      I donot agree. See third item of the sub-range novelty test: "the selected range is not an arbitrary specimen of the prior art, i.e. not a mere embodiment of the prior art, but another invention (purposive selection, new technical teaching)." So, the effect for purposive selection must be a technical teaching.
      Furthermore, need a technical effect to make it inventive.

      Delete
  6. I took and passed paper D last year and, prima facie, I can say this year DII was a lot harder.

    Altough a standard DII does not exist, this was not a DII format one could reasonably expects, especially for a candidate who acts under time pressure.

    To me, DII should focus more on strategy, what your clients owns, what his competitors owns and what can be done to improve the client position.

    Altough Q1 was related to a client's asset (success of opposition), Q1 tested legal aspects, (admissibility, late notification, reformatio in peius, party by right) and should have been put in DI.

    Just my 2 cent.


    Observer

    ReplyDelete
    Replies
    1. I agree it was not expected, as none of the 5h D exams had big legal cases in the DII.
      However, there were legal cases of similar nature and size in earlier DIIs, e.g, par.[011] ff. of the DII 2012 (you get find the original version via http://www.epo.org/learning-events/eqe/compendium/D/DII.html), and I donot think the REE/IPREE changed w.r.t. the scope of what is tested in the D paper. So moving a big legal topic in the DII every once in a while was done in the past.
      So, yes, unexpected, but not unprecedented.

      Delete
  7. Dear Delta Team:

    Thanks for the detailed solution.

    >Range 155-250 deg is disclosed in PCT-FK1.
    >below 200 deg: less Acryl...
    >Preferred range in PCT-BB2: 175-200.

    "If the effect found in the tests as described in [006] is anyhow disclosed in PCT-BB2, we can argue that..."

    I don´t see any requirement of an original disclosure of a technical effect. The advantageous effect is given - even if not originally disclosed. Therefore, we do not deal with a problem of substantive law here but with the procedual question of presenting evidence. Such evidence is available (test results) and can be filed even after filing (an exception may only be the late filing of experimental results in the case of a medical use claim).

    Hence: withdrawing PCT-BB2 would result in the loss of the patententable process claims.

    All in all, a very difficult and confusing part DII, since there were no clear-cut solutions/answers to many of the problems raised (appeal admissible or not, numerical ranges, PCT-BB2: combination(!) of trumpet from drawing with example novelty destroying or only not inventive). This made it very hard to put the final picture together.

    ReplyDelete
    Replies
    1. "The advantageous effect is given - even if not originally disclosed."

      Although not described in the Guidelines explicitly for purposive selection, it is well-described for inventive step that:

      GL (2016) G-VII, 5.2:
      "As a matter of principle any effect provided by the invention may be used as a basis for the reformulation of the technical problem, as long as said effect is derivable from the application as filed (see T 386/89). It is also possible to rely on new effects submitted subsequently during the proceedings by the applicant, provided that the skilled person would recognise these effects as implied by or related to the technical problem initially suggested (see G-VII, 11 and T 184/82)."

      Delete
    2. Dear Roel:
      I do not see too much of a problem with the technical effect:

      - T 386/89 is rather concerned with the complete reformulation of a problem to be solved (not the case here)
      - The paper at least implies that the effect was described and that at least the problem of acrylamide content was mentioned
      - There is also no explicit indication that the effect of the trumpet nozzel is described in EP-BB2 or EP-BB3
      Hence, none of the subject-matter in PCT-BB2/EP-BB3 would be inventive
      - I do not think that it was intended/expected to discuss both approaches in the answer

      Delete
    3. Hi Anonymous

      - Yes, it is a reformulation, as the purposive selection requires a different or stronger effect that the original range - so, the objective technical problem needs to be reformulated from what it was initially to what is is with the differnt or with the stronger effect.

      - How does the paper imply it? I donot see it, not in a "directly and umambiguously" or "inevitable" sense. On the contrary: for EP-BB1 and PCT-FK1 it does explicitly tell us that effects are included. So not mentioning them for EP-BB2 and PCT-BB2 means that they are not included.

      - That is true. So also for the trumpet there is a problem, -in that case- with inventive step. The DeltaPatents answer is not consistent in this, maybe Roel can indicate why. Or maybe actually it is, as they mention in the alternative solution that they also refile for the trumpet and then also include the advantageous effect ot that.

      - If not expected to discuss both, then only approach is the one based on the facts in the papers, i.e., using the facts that the applications have no effects mentioned: sub-range not novel, trumpet not inventive.

      I agree it is very very unlikely that the exam committee meant to exclude the effects from the applications, but the paper is not what they intended but what it is. The IPREE is very clear about that too.

      So, I see big problems. And a "solution" could be that they admit that they had a design error in the exam and give full marks for solutions that handled the applications as if the effects were in them (as intended) as well as for solutions that handled the applications as they were presented in the paper so without the effects in it for the range, the sub-range, and the trumpet. Candidates cannot know what was intended, but have to use the facts, so shall get full marks if they use just the facts - Rule 2293) IPREE. Candidates that understand what was intended should also not be punished for that, as the exam committee themselves did not do any different.

      Will be interesting to see what happens in the near future. Will the exam committee or board post a message to get us all out of the figgy situation we are in now? Or will the papers just be marked with one marking sheet, not douing justice to the quality of the answers that went into the other direction, and will we see many appeals, which will in part be succesful? Or?

      Tom

      Delete
    4. Thx Tom for answering the points raised by "Anonymous 2 March 2018 at 14:07" - I agree with your first 3 dashes.
      As to the 4th dash, I indicated already why we included both approaches. Whether both or one were expected, and if only one, which one, remains to be seen.

      Delete
    5. This comment has been removed by the author.

      Delete
    6. OM:

      -How does the paper imply it? I donot see it, not in a "directly and umambiguously" or "inevitable"

      > True - the paper could have been drafted much clearer in this regard. At least during the Examination, I just did not see so much of a problem in this regard because otherwise the complete exam would not have made any sense to me.

      >For "EP-BB1" - well there is a technical effect in the claims (which actually should have resulted in a lack of clarity problem in examination proceedings). However, there is no prove /experimental data described as contained in the application that a certain fruit combination actually results in the acyl level indicated - even for claim 3 + 2 + 1 (for claims 1 and 2 it is apparently not available). Nevertheless, claims 3 +2 + 1 seem grantable here.

      Peter

      Delete
    7. Hi Peter,

      "At least during the Examination, I just did not see so much of a problem in this regard because otherwise the complete exam would not have made any sense to me"

      This is a practical approach in the exam (and also for us one of our reasons to include it as a scenario in our answer), but not really a valid legal argument as to why it would be like that...

      Our "alternative soution" is the result of using the descriptions of EP-BB2, PCT-BB2 and EP-BB3 as they are presented, i.e. w/o effects of the range, the sub-range and the trumpet. Then, a solution to withdraw all nonpatentable applications and filing a new one that also includes the effects for the invention that is still novel as not public yet (apparatus w mixer and trumplet nozzle) (the Newspaper killing the range-invention) is a sound solution.

      Delete
  8. jesus what messy questions, was this intentional or accidental. I see an increase in the EQE appeals coming ... At least I was not completely wrong ...

    ReplyDelete
  9. This one was tough. Almost feels designed to break the candidates’ self-confidence. I am used to the solutions jumping out at me, not this mess. Starting out with a very complicated appeal, where I knew I was in trouble, did nothing for the self confidence in answering the rest. Then running into uncertainties in the text, such as not finding anything written specifically about the later applications actually mentioning acrylamide levels, although it says that investigation into temperature dependencies lead to the research behind the later applications, made me feel even further into deep waters – I thought there was something I did not see, not that it was written to be unclear ☹. So, panic ensued, and valuable time was wasted getting myself back under control and looking for information that was not there. The last question did not help, it seemed too easy, giving standard reasons for why one would want the patent (except the part of having admitted to infringement in the newspaper article), so by then I really thought there had to be things I was simply missing, and again spent valuable time hunting for something that was not there. Sigh. I am my own worst enemy, it would seem.

    ReplyDelete
    Replies
    1. Hi Anja,
      As you may have grasped from our answer, some of us were also puzzled by similar things as what you described with "Then running into uncertainties in the text, such as not finding anything written specifically about the later applications actually mentioning acrylamide levels, although it says that investigation into temperature dependencies lead to the research behind the later applications, made me feel even further into deep waters – I thought there was something I did not see, not that it was written to be unclear". It especially made me worry as there was a lot of valuable information (test results, effects) in [006] and [009] first sentence, of which it was however not indicated that that information was also included in the applications of the client, which therefore seemed to miss any effect. Effects were however indicated in EP-BB1 and PCT-FK1. In earlier papers, applications always mentioned effects (to be confirmed), either explicitly, or indirectly with phrases as "we included all these results and advantages in EP1", "EP2 has the same text as EP1, but also includes ...".
      What did you do in the end? Discuss both "no effect mentioned, so not patentable; but, if anyhow included or late-filed effects accepted, then patentable" as we did in our answer, or choose one direction only?

      Delete
  10. Hope this is no double posting now. Was wondering if EP-BB3 could be regarded as first application for mixer+trumpet as EB-BB2 was deemed withdrawn before filing EP-BB3, was not open to public and did not serve as priority in PCT-BB2 for mixer+trumpet as PCT-BB2 could not validly claim priority as mixer+trumpet+screw is disclosed. So, if EP-BB3 would be the first application one could add an undisclosed disclaimer to screw to make it novel over PCT-BB2, file a PCT-BB3 with one claim to mixer+trumpet without screw priority EP-BB3 and one claim mixer+trumpet+screw priority PCT-BB2. Unfortunately could not find any details in case law about the priority situation and the German text of a87(4) might be misleading. Best

    ReplyDelete
    Replies
    1. Good question. You mean that the "must" in PCT-BB2 has the effect that the skilled person would read any "mixer + trumpet" in PCT-BB2 as "mixer + trumpet + screw", i.e., as a different invention than what EP-BB2 showed? And that thus the priority claim to EP-BB2 is not valid as that only has the more generic "mixer + trumpet"? And that therefor the Art.87(4) should not fail? I am not 100% sure, but I donot think that helps to discount the Art.87(4)-requirement that EP-BB2 should not have served as a priority for PCT-BB2, which I have always seen as a procedural requirement to the application (i.e. that it is used as a priorty document as such), and not as a substantive requirement to the subject-matter in it (i.e. the validity of piority of specific subject-matter), because also the reference to "withdrawn, deemed to be withdrawn without being open to public inspection" relates to an application as such and not to individual, differnt subject-matter in it.

      But irrespective of that: Ar.t87(4) is of no help: you seem to overlook that when EP-BB3 was filed, EP-BB2 still had its priority right outstanding: EP-BB2 was deemed to be withdrawn, but its priorioty right was not withdrawn [Art.87(3)+Art.87(2) EPC]. So, you cannot consider EP-BB3 as the first application under Ar.87(4).

      Delete
    2. Thanks for the comprehensive reply. Agree that EP-BB2 served as valid priority for PCT-BB2 for the process; but then it came to the formulation of German A87(4) that puzzled me, there is mentioned "the" priority and not "a" which somehow implies for me that the priority for the subject matter in that application has to be "validly" claimed. Validly because I do see priority in this aspect as an substantive requirement, because the right to priority should be granted for the first invention as a matter of fact and should not be bound to any procedural aspects which would somehow deprive the applicant from what he/she has created. As no valid priority claim from EP-BB2 for mixer -> EP-BB3 first application for mixer

      Delete
  11. Part II was a mess this year. Do they take difficulty into account when marking?

    ReplyDelete
  12. I also had a hard time with the D2 part.

    Concerning the improvement for PCT-BB2 with the temperature ranges I finally settled with a (undisclosed) disclaimer to exclude fruit D and keep the whole range 150-220 C.
    The only effect mentioned was tied to the 220 C, so I saw no reason/basis to adapt the range to the preferred subrange which had no effect apparently. By excluding the fruit D I thought to reestablish novelty, which would be enough. But after reading the answers here I am doubting ...

    ReplyDelete
    Replies
    1. Why can you do an undisclosed disclaimer? E(URO-)PCT-FK1 is not 54(3).

      Delete
  13. @Roel

    From the above discussion I could not get why the subject-matter of the combination claims 1+2 (patent as amended after opposition), could be considered for Examination by the Board of Appeal. The opposition was filed specifically only against claim 1 alone (the Opponent argued against claim 1 without any other fruit p [3] of the paper) and on the ground of A. 100 (b), i.e. 1+2 is unopposed subject-matter.

    I am aware of the G 1/91. However, this decision only (and specifically) relates to patentability (see Case Law of the Boards of Appeal E 3.2.1 c)), that is, according my understanding A. 100 (a). In our case we have 100 (b).

    On the other Hand the patentee was appealing against a subject-matter which got lost in the opposition even if unopposed (opposition only against claim 1 and not against 1+3).

    Thank you very much for your help, and also thanks for the blog it is really instructive!

    ReplyDelete
    Replies
    1. We had a discussion about this. We concluded that the phrase "The opponent attacked the patent" was probably to be interpreted to "the patent as a whole", so all 3 claims, as it did not explicitly say that the extent of opposition was limited to claim 1 only.
      But even if it was, then G 9/91 (r.11; hn) allows to attack claim 2 as well, as G 9/91 says "However, subject-matters of claims depending on an independent claim, which falls in opposition or appeal proceedings, may be examined as to their patentability even if they have not been explicitly opposed, provided their validity is prima facie in doubt on the basis of already available information." It may be argued that an insufficiency problem of claim 1 out the sufficiency of disclosure of the subject-matter of claim 2 prima facie in doubt on the basis of already available information.

      Delete
    2. Thank you very much for your quick answer!

      I am adding the following considerations.

      According to my understanding of P [3] the opponent showed that a fried ball without (emphasis added) any other fruits ( English version is not published yet but the German goes:"unter Verwendung von Frucht A und ohne andere getrocknete Früchte") had an enhanced content of Acrylamide.

      Which would point in the direction of claim 1 alone without any other fruit.

      From your wording, G 9/91 relates again to patentability and not A. 100(b) EPC.

      Also it was necessary to perform other tests for both the patentee and the opponent to understand that that claim 1+2 did not meet A 100 (b) EPC (see p[4]) and also p [6]) which would point in the direction of no prima facie obviousness.

      Were you referring to a section of the blog when mentioning you had a discussion about this?

      If so my apologies for not having spotted it.

      Thanks again!

      Delete
    3. > Were you referring to a section of the blog when mentioning you had a discussion about this?
      No, a discussion we had when discussing the answers. One of us initially had some doubt about what the extent was.

      > G 9/91 relates again to patentability and not A. 100(b) EPC.
      But why would it be different for enablement than for novelty, or, if you wish, for Art.100(b) than for Art.100(a)?

      By the way, an English version is now accessible via the link in our blog, just before our "signature". It says in [003]: "...according to claim 1 using fruit A and no other dried fruit had a higher level of ..."
      (At this moment, the Compendium pages link to the DE version from the EN page).

      Delete
    4. G 9/91 refers to "patentability", not specifically to "Art.100(a)". I did not check in the earlier decision whether such narrow interpretation weas meant, but I cannot derive that from G 9/912 itself. I see no reason why it would not apply to all Art.100 grounds.

      G 9/91, r.8 and r.10-11:
      8. As to the former requirement, which is the subject of the referral in case G 9/91, it has to be noted that in practice it is rather unusual that the opposition is limited to only a certain part (subject-matter) of the patent. Normally, the whole of the patent is opposed. Sometimes this is not explicitly explained in the notice of opposition but it appears by implication that the opposition is directed to the whole of the patent. There may be cases when the notice of opposition has to be interpreted in this respect, in particular when only certain claims are explicitly dealt with in the notice (cf. the decision of 20 July 1989 in case T 192/88). However, the Enlarged Board sees no reason in the present context to go further into this problem, but will confine its considerations mainly to the situation where it is clear from the statement under Rule 55(c) EPC that the patent is opposed only to a certain extent, as illustrated e.g. by the case before the referring Board of Appeal 3.3.3 (see paragraph I above; cf. also paragraph 11 below).

      10. The conclusion of the Enlarged Board on this matter comes close to that of the above decision in case T 9/87. The requirement of Rule 55(c) EPC to specify the extent to which the patent is opposed within the time limit prescribed by Article 99(1) EPC would obviously be pointless if later on other parts of the patent than those so opposed could freely be drawn into the proceedings. This would also be contrary to the basic concept of post-grant opposition under the EPC as outlined above. By limiting the extent to which the patent is opposed to only certain subject-matters, the opponent deliberately refrains from making use of his right under the EPC to oppose remaining subject-matters covered by the patent. Such subject-matters are therefore, strictly speaking, not subject to any "opposition" in the sense of Articles 101 and 102 EPC, nor are there any "proceedings" in the sense of Articles 114 and 115 EPC in existence concerning such non-opposed subject-matters. Consequently, the EPO has no competence to deal with them at all.

      11. It follows that the answer to the first question put to the Enlarged Board in case G 9/91 has to be affirmative. However, even if the opposition is explicitly directed only to the subject-matter of an independent claim of a European patent, subject-matters covered by claims which depend on such an independent claim may also be examined as to patentability, if the independent claim falls in opposition or appeal proceedings, provided their validity is prima facie in doubt on the basis of already available information (cf. T 293/88, OJ EPO 1992, 220). Such dependent subject-matters have to be considered as being implicitly covered by the statement under Rule 55(c) EPC (cf. paragraph 8 above).

      Delete
    5. >But why would it be different for enablement than for novelty, or, if you wish, for Art.100(b) than for Art.100(a)?

      Because the Case Law of the Boards of Appeal E 3.2.1 c) makes clear that there is only one exception for unopposed subject-matter which can be considered by the board of appeal, that is the G 9/91 for patentability.

      A 100 (b) is not considered by the Board in that decision, therefore should fall in the standard case (I assume), i.e. unopposed. At this stage I am unable to derive any other conclusions in absence of other case Law.

      For completeness I am copy pasting:


      c) Unopposed subject-matter not reviewed

      No part of a patent's subject-matter (e.g. individual claims) not opposed within the nine-month time limit can be reviewed in either opposition or appeal proceedings. The opponent's statement under R. 76(2)(c) EPC (R. 55(c) EPC 1973) establishes the extent to which the patent is contested and thus the formal competence of the opposition division or board of appeal. The only exception to this principle concerns an opposition explicitly directed only to the subject-matter of an independent claim. In such a case, subject-matter covered by claims which depend on the independent claim can also be examined as to patentability (see e.g. G 9/91, OJ 1993, 408; see also T 323/94).

      > Also further tests indicates, in my opinion, that it was not prima facie relevant

      Matteo

      Delete
    6. But the cited section refers to “patentability”, not “Art.100(a)”-grounds. And also enablement is a requirement for an invention to be patentable. I cannot see that a limitation was intended to Art.100(a) in G 9/91.
      Interesting discussion!

      Delete
    7. @ Roel

      Just out of completeness and to finish setting out my line of thoughts:

      The G 9/91 only deals with 100(a) and also under item 11, when putting forward the patentability criterion, makes reference to the T 293/88 which is a decision once again only pertaining inventive step.

      >But the cited section refers to “patentability”, not “Art.100(a)”-grounds. And also enablement is a requirement for an invention to be patentable.

      From the EPC (Articles) we get that the chapter PATENTABILITY only refers to Articles 52-57 (it is basically the title of Chapter I Part II). From the EPC thus we have direct basis for understanding that patentability means Articles 52-57. Enablement is under the "Filing and requirements of the European patent application" section I could not find another definition so far (I have found something in the GL, but not being a legally binding text I am quite reluctant to use it for supporting my arguments).

      That is, I am still not persuaded that enablement would be comprised in the general meaning of the word patentability. Please let me know if you see it differently.

      Moreover

      Also the paper goes:

      "The opponent attacked the patent under Article 1OO(b) EPC, demonstrating that a fried ball of dough prepared according to claim 1 using fruit A AND NO OTHER (emphasis added) dried fruit had a higher level of acrylamide than claimed". (The German version use a stronger wording I would say by using the word "indem", I am neither EN nor DE mother tongue but to be on the safe side i checked both Versions during the Exam).

      That is, from than excerpt of the paper above it is prima facie obvious that the objection raised by the opponent does not apply (by definition) to the dependent claims as they all comprise more than A.

      Finally I have still the old argument of the tests that were necessary and point once again to the absence of possible prima facie considerations within the meaning of G 9/91.

      In summary, according to my present understanding.

      1) I am not sure the G 9/91 finds application for a A. 100(b) EPC in the first place;
      2) Even if it does, the prima facie criterion is not fulfilled because fo the type of opponents attack ("and no other fruit");
      3) The tests again points out that the issue could have not been easily considered by the Opposition Division or BoA on their own motion.

      On the other hand by referring to the Statements made during the opposition the patentee was also incorporating defence for 1+3 that, though never attacked by the opponent, got lost in the opposition for no reason. For the same reason as 1+2 overcome the objections, also 1+3 would overcome the objections, that is, the attack of the opponent is deprived of substantiation, as it only relates to A alone.

      Interesting discussion indeed! at least for me, I am learning a lot.

      Well it seems we have different opinion about this, I am looking forward to being able to read the Examiner's report.

      Thanks again,
      Matteo

      Delete
    8. Hi Matteo,

      Good reasoning, interesting discussion. We will not converge to one solution I think, but let me give my response to your comments:

      As to G 9/91: I understand your reasoning. I see however no reason to erad in G 991 that it was meant to relate just to 52-57. And it it did, it could be applied mutatis mutandis to enabkement - same kind of arguments hold why the extent could extend to the dependent claims as for 52-57.

      As to your moreover: good points. Prima facie argument is maybe not rock solid, maybe not even correct. Moreover, what puzzles me is that [003]&[004] does not indicate what the first instance submissions are w.r.t. the allowabiloty of claim 2 - if there was no doubt raised as to enablement of claim 2, it is something that "could have been submitted at first instance, but was not" and for that reason an objection agaimst claim 2 in appeal could be considered inadmissible under R.12(4) RPBA. We donot have enough information to judge the later, as it is at the discretion of the Board, so we cannot draw a firm conclusion on it - were we expected to discuss both sides?

      Looks like we agree that what needs to be addressed in an answer to Q.1 is:
      1) extent - what was it, can it be extended to dependent claims (G 9/91)?
      2) admit and allow new request from opponent to insufficiency of claim 2 - same ground, although to different claim, but new facts (allowed T 611/90; R.12(4) RPBA?)
      3) prima facie doubt as to validity of claim 2 - in view of opposition based on tests on A w/o other fruit?
      4) proprietor party of part-as-of-right due to mere reference to first instance?
      5) what were the first instance submissions?
      6) admit and allow new request from proprietor to amend claims to 1+3 or 1+2=3?
      7) admit and allow new evidence from prioprietor that 1_3 will work - and how does this effect admissavlity and allowabiloty of request to 1+2+3?
      I find it difficult to find all the facts needed to address each of these points - so to come to a conclusion... I feel sympathy for your arguments, but you also use information that is not available in [003]-[005], so I donot (yet) want to change our answer.

      We will need to see what the Examiners' Report will say, and whether we can understand how the conclosion therein is reached from the information present in [003[-[005] and the relevant case law.

      Would be interesting to hear more opinions and especially argumentation (so, a full answer to DII Q.1) from other candidates and tutors. Did we and/or Matteo overlook / misinteprete / inter[rete differently information from the paper than you did? Please comment.

      NB: The Rule of Procedure of the Boards of Appeal (RPBA), esp. Rule 12-15, are not yet part of the EQE Syllabus. However, as they reflect in the Case Law that is part of it, it seemd fully justified to me that candidates have some knowledge of Rule 12-15 RPBA and/or can apply their principles. In DII, no legal citations are needed, so no specific reference to a RPBA provision is needed. Also be informed that at the Meeting of the Exam Committees with the tutors in last September (see epi Informatiom 4/2017), the tutots expresse their almost unanimous support to inclde the RPBA in the EQE syllabus and to test it.

      Delete
    9. If I understand this decision correctly, according to T 907/03, reason 2, all grounds of opposition can be considered when extending the extent from an independent to a dependent claim:

      2. Umfang der Prüfung

      Im Einspruchsverfahren war nur Anspruch 1 des Streitpatents angegriffen worden. Entgegen der Auffassung der Beschwerdeführerin II sieht sich die Kammer jedoch befugt, alle Ansprüche sowohl des Haupt- als auch des Hilfsantrags zu prüfen, also auch die Alternativen der Ansprüche 10 bis 14 gemäß Hilfsantrag, die der erteilten Fassung dieser Ansprüche entsprechen (Ausnahme: Klarheit, siehe unten Punkt 4.2.5). Dies folgt aus der Entscheidung G 9/91 (ABl. EPA 1993, 408; vgl. Punkt 11 der Entscheidungsgründe) und der in der Entscheidung T 525/96 (nicht veröffentlicht; vgl. Punkte 4.3 bis 4.6 der Entscheidungsgründe) getroffenen Auslegung dieser Entscheidung der Großen Beschwerdekammer.

      http://www.epo.org/law-practice/case-law-appeals/recent/t030907du1.html

      Delete
    10. Hi Roel,

      Thank you very much again for your post, it is getting more and more interesting, I am trying in the following to address all the points you raised.

      A) >As to G 9/91: I understand your reasoning. I see however no reason to erad in G 991 that it was meant to relate just to 52-57.

      Here I was hoping you could indicate legal basis for the patentability definition you are using: if on the one hand patentability= A. 52 to 57 is directly indicated by the EPC, patentability also encompassing enablement is not. I am aware that in some instances the patentability was broadened,(for instance A. 115 EPC) but this is quite an exception and does not apply to the general case according to my understanding (in A. 115 patentability also encompasses clarity for instance).

      I think legal support on your side would greatly help to follow your line of arguments. Right now in the absence of it I am having really hard times...

      B) > And it it did, it could be applied mutatis mutandis to enabkement - same kind of arguments hold why the extent could extend to the dependent claims as for 52-57.

      Do you think we would be allowed to or expected to make such considerations? My understanding of this is that either enablement is encompassed by the G decision, or we would have to wait for another G or T decision, to help us determining whether this could be applied “mutatis mutandis”. I suppose this task rests only with a board. (But this is just a side consideration.)


      C) As to your bullet list, and using the same numbering:

      1)-3) We need to be sure about the patentability definition see A).
      4) In connection with the “mere reference to first instance submissions” we have to remember that (provided my understanding of the G 9/91 is not that much off the mark) there is also case law in which a reference was indeed accepted: in the case in which it already adequately addressed the grounds underlying the contested decision (T 355/86, T 140/88, T 216/10 Case Law Book (8th edition, IV-E, 2.6.4 a)).

      Again a claim set including 1+3 overcomes all (meaning the only one) objection raised in the first instance by definition (1+3 is more than A and the opposition objection is automatically mooted). Just on the basis of this at least a portion of the Proprietor request is allowable on the face of it (subject-matter undisputed). So actually, in a sense it does duly address.

      5) As to the first instance submission, for my reasoning to hold, all we need to know is that opponent was attacking ONLY claim 1 and defendant was defending ALL the patent as granted, in this way he was also defending 1+3, note that having this claim combination never been attacked, defendant needs not to argue in favour of compliance with the EPC, it is undisputed. We need no other details of the first instance submissions and the above information we can derive from the paper.

      6)-7) Need to address the above.

      E) Yes I totally agree that we could really use some fresh opinions to overcome the deadlock. I am genuinely interested in understanding these points and not just trying to make my case here.

      F) I am afraid I cannot derive the conclusion that “all the grounds of opposition can be considered from the T 907/03”: the excerpt you are quoting only states that even though only claim 1 was attacked (and it was attacked on the basis of 100(a) EPC, see I. second paragraph) the board also examines all the other claims of the Main and Aux req. But I again I am no mother tongue.

      Really not sure we will converge, but I would greatly appreciate if you could at least address A) as this appears to be the core of the divergence.

      Thank you very much for your time and sorry for the extreme lengthy post,
      Matteo

      Delete
    11. @A-B): See below for my comment "Roel van Woudenberg 5 March 2018 at 10:15"

      @C): Case Law Book (8th edition) IV-E, 2.6.4 a): "In a small number of isolated decisions, however, the boards have accepted a general reference to submissions at first instance as potentially constituting grounds for an admissible appeal (T 355/86, T 140/88, T 216/10), but these decisions were given in special cases in which those submissions already adequately addressed the grounds underlying the contested decision."

      So, IT the proprietor would have adequeately addressed the invalidity of the opponent's arguments to the insufficiency of original claim 1 and the validity of claim 2 in his first instance submissions before the opposition division, it could have been sufficient. However, it is not described that he did and in view of [005] he agrees that original claim 1 was not sufficient disclosed, and he need to fall back to 1+3. Further, he did not indicate "how the decision should be amended". So, I donot see how he could be one of these rare exceptions.

      Delete
    12. > @A-B): See below for my comment "Roel van Woudenberg 5 March 2018 at 10:15"

      Roel, I believe you may have overlooked point F) above (Anonymous 5 March 2018 at 21:37) in which I was specifically addressing your previous post and the decision you mentioned ��. Also for a quick English reference of that decision (T 907/03) please check "Case Law of the Boards of Appeal IV D 3.2 Extent of the opposition", in particular second paragraph last line: that that decision merely relates to the extension (in view of G 9/91) of an objection to claim 1 also to Aux Req (and of course Main Req).

      I am afraid we cannot derive the conclusion that “all the grounds of opposition can be considered” from the decision you are citing and especially from the paragraph you are quoting.

      As currently understand it, we still have no legal support for your broad use of G 9/91.

      >I feel sympathy for your arguments, but you also use information that is not available in….

      At this point I have to admit I am quite puzzled: in view of [3] of paper D it is clear that only claim 1 is attacked (see our above discussion) and G 9/91 specifically requires patentability, which we could not justify so far.

      Were you maybe meaning referring to another decisions? Is there anything else which we could use to further provide legal support to your reasoning? Or is there anything I am missing/ not duly considering?

      Since this is a major point of attention I am not addressing the other issues of this thread until we have, possibly agreed that dependent claims can or cannot be attacked.

      Please pardon my insistence but to be in a position to follow your reasoning we really need legal basis for an attack to the dependent claims.

      Thank you very much and have a nice day,
      Matteo

      Delete
    13. opps seems like the blog does not support the smileys
      MAtteo

      Delete
    14. Hi Matteo,
      You wrote "As currently understand it, we still have no legal support for your broad use of G 9/91.".
      I admit that I did not find explicit legal basis that says that dependent claims can be in the extent of an opposition filed w.r.t. only the independent claim if the only ground is insufficiency of disclosure. However, there is -as I said before- in my view nothing in G 9/91 which deliberately limits the concept of G 9/91 to only Art.52-57, and its concept can in my view also be applied to insufficiency: the reasoning why the extent can extend to the dependent claims uses nothing specific for Art.52-57 and the arguments given why it can extend to the dependent claims apply mutatis mutandis for insufficiency - in my view.
      A reason that there is no case law can be that it never gave a dispute so no appeals were filed on this, that insufficiency is in general hard to overcome, that oppositions are only very rarely filed w.r.t. only the independent claims, that appeals were filed but interlocutory revision was applied in all those cases -- such that irrespective of whether it can be extended to dependent claims for insufficieny or not, there is no case law to support one or the other view.

      I understood r.2 from T 907/03:
      "2. Umfang der Prüfung

      Im Einspruchsverfahren war nur Anspruch 1 des Streitpatents angegriffen worden. Entgegen der Auffassung der Beschwerdeführerin II sieht sich die Kammer jedoch befugt, alle Ansprüche sowohl des Haupt- als auch des Hilfsantrags zu prüfen, also auch die Alternativen der Ansprüche 10 bis 14 gemäß Hilfsantrag, die der erteilten Fassung dieser Ansprüche entsprechen (Ausnahme: Klarheit, siehe unten Punkt 4.2.5). Dies folgt aus der Entscheidung G 9/91 (ABl. EPA 1993, 408; vgl. Punkt 11 der Entscheidungsgründe) und der in der Entscheidung T 525/96 (nicht veröffentlicht; vgl. Punkte 4.3 bis 4.6 der Entscheidungsgründe) getroffenen Auslegung dieser Entscheidung der Großen Beschwerdekammer."
      as
      The extent of opposition was only claim 1 of the main request. However, the division considers itself competent to also examine the other claims of the main request )...), except for clarity. This follows from G 9/91 and T 525/96.
      Thus, also all dependent claims can be examined for all grounds of opposition, but not for clarity. I.e., the extent extends to all dependent claims AND examination is not limited to Art.52-57 but to all opposition grounds.

      [continues in next comment]

      Delete
    15. [continued]

      Note that T 525/96, reason 4.6, also extends the scope of G 9/91, albeit in another way: to another INdependent claim:
      T 525/96, 4.6 "The present case is slightly different since the claim under consideration, new claim 13, is not a dependent claim but an independent product-by-process claim based on a process claim which failed during the appeal proceedings. As the product protected by this claim is not defined by any actual product feature but simply by way of its preparation process, its patentability can only be justified by an allegedly novel and inventive feature derived from this process. Therefore, in the board's view, the relationship between claim 13 and the original process claims (13 and 14 [the extet of opoosition was just these 2 process claims]) for the assessment of patentability is even stronger than that between independent and dependent claims referred to in the decision of the Enlarged Board of Appeal, since the invalidity of the product-by-process claim follows directly from the invalidity of the process claim. Hence it is justified to apply the conclusion of the Enlarged Board of Appeal in G 9/91 also to the present situation."

      Using those words, to me is seems justified to apply the conclusion of the Enlarged Board of Appeal in G 9/91 also to the present situation, such that the extent of opposition can be extended to the dependent claims if the only ground is insufficiency Art.100(b)/Art.83 and not only for Art.100(a)/Art.52-57.

      Note: it is to be reminded that the G 9/91 principle of extending to the dependent claims is subject to a strong condition: In point 11 of G 9/91, the Enlarged Board of Appeal laid down that "even if the opposition is explicitly directed only to the subject-matter of an independent claim of a European patent, subject-matters covered by claims which depend on such an independent claim may also be examined as to patentability, **if the independent claim falls in opposition or appeal proceedings, provided their validity is prima facie in doubt on the basis of already available information**. Such dependent subject-matters have to be considered as being implicitly covered by the statement under Rule 55(c) EPC1973 [= Rule 76(2)(c) EPC2000]."

      Delete
    16. Hi Matteo,

      GL (2016) D-V, 2 reads:
      2. Extent of the examination
      2.1 Extent to which the patent is opposed
      In the unusual case where an opposition is limited to only a certain part of the patent, the Opposition Division has to limit its examination to the part opposed. However, if the opposition is directed only to an independent claim, the dependent claims are considered to be implicitly covered by the extent of the opposition and may be examined by the Opposition Division, provided their validity is prima facie in doubt on the basis of the information already available (see G 9/91). Similarly, if only a process claim is opposed, a product-by-process claim making reference to the same process is considered to be implicitly covered by the extent of opposition and may be examined under the same conditions as above (see T 525/96).

      This paragraph uses just the words "may be examined", without any limitation to "patentability".

      So, legal basis could be cited (although not needed in DII) as G 9/91 + GL (2016) D-V, 2.1 for the argument that the dependent claims canalso considered to be covered by an opposition only directed to the independent claim, for any opposition ground, if in prima facie doubt.

      Delete
    17. Interesting discussion!

      But still leaves open important issues for Q.1 of the paper:

      1) was the extent of opposition originally just claim 1 as suggested by [003] where only insufficiency of claim 1 was demonstrated? Or was it the patent as a whole, as suggested by "attacked the patent" in [003]?
      The first seems most likely with the facts as they are presented in the paper, as no comments are made at all about what was submitted w.r.t. claims 2 and 3 in first instance. And claim 2 can only be maintained without any arguments if there was no discussion allowed on it. Or?

      2) Claim 1 is not sufficientely disclosed because, as demonstrated in the opposition, a friend ball prepared using the method of claim 1 using ONLY A AND NO OTHER FRUIT had a higher level of acrylamide than claimed. Does that raised prima facie doubt to the insufficiency of claim 2 (...fruit COMPRISING A AND B) based on the already available information, which is the evidence w.r.t. A-ONLY? No information given. No reason why it would be.

      3) What will the Boards let in and what not based on their rule "what could have been submitted in first instance should have been submitted in first instance"? In other words: do we have all facts that we need to be able to conclude what the Boards will admit? Especially also in view of the proprietor being a party as of right?

      Roel, I know you addressed the above in your answer, but I am not convinced. Discussing various sides to deal with the uncertainty seems a too high burden - would get this Q.1 to 20-25 marks if I estimate time needed relative to rest of DII. I would be interested to hear from others what they think.

      TK

      Delete
    18. Thank you for your detailed post about Q1.
      I also considered that the Opposition was limited to the first independant claim, since there was no prima faciae evidence (G9/91 + "no other dried fruit" in [003]) that claim 2 or claim 3 could not satisfy Art83. I even considered as a proof the fact that claim 2 has been allowed by the Opposision Division, and that we do not have any information that this claim has been discussed by the Opponent.
      Hoping others have made similar considerations...
      KB

      Delete
    19. Good Day, Roel and thanks again for your help, I greatly appreciate it! Here below my comments to the points you raised.

      >However, there is -as I said before- in my view nothing in G 9/91 which deliberately limits the concept of G 9/91 to only Art.52-57.

      Actually, the Board when taking the decision EXPLICITLY decided to use the word patentability which has a very precise meaning within the EPC, that is A. 52-57 EPC (see corresponding chapter of the EPC).

      Therefore, I have to traverse your opinion on this point.

      >reasoning can be applied to….

      I also have to rebut this point I am afraid. Whether or not this reasoning can be used, cannot be decided by a candidate sitting the EQE, the task of providing guidance as to how interpret the law (or the case Law if needed) rests only with the Board of Appeal.

      As I currently understand this, we can take the teaching of a given decision which can then be applied as long as the legal framework stays the same. Changing the legal framework (for instance A. 100(a) to A. 100(b)) may result in a whole new ball game, which a Board would have to consider, I would say.

      >I understood r.2 from T 907/03

      In that decision the Board stayed within the framework of patentability (A. 100 (a)). They did not went outside it. Therefore, it does not teach (or imply) to broaden the attacks from A. 100(a) to A. 100(b) (or to A. 100(c), clarity is in any case not a ground for opposition).

      Same reasoning would apply mutatis mutandis to all the other decisions you cited. (Please note that for instance also T 525/96 refers explicitly to patentability, i.e. again within the legal framework set by G 9/91).

      It is my understanding that by trying to decide what could be justified or not we are substituting ourselves to the Board. Therefore, I am not going too much into the details of how the decisions you cited could (or could not) support a possible 100(b) extension of the G 9/91 (of course in real life and representing the opponent, I would have gone down this path, trying to bring up good arguments for the board’s attention).

      > So, legal basis could be cited (although not needed in DII) as G 9/91 + GL (2016) D-V, 2.1

      I am afraid I am also in no position to follow this one:

      i) GL are not a legal text or legally binding strictu sensu, Case Law of the Board of Appeal III, S, 2 goes: "The Guidelines are intended to cover normal occurrences. They should therefore be considered only as general instructions.

      ii) Also consider the Case Law of the Board of Appeal III, S, 1 which goes: The Guidelines for Examination are not binding on the boards of appeal (settled case law; see, for instance, T 1561/05 of 17 October 2006, point 1.5 of the Reasons).

      That is, I currently cannot follow that a G decision is interpreted in light of the Guidelines, actually is the other way round; Guidelines should be interpreted in light of the corresponding G decision (if applicable).

      Delete
    20. Since from paragraph [3] it follows that only claim 1 was attacked and it would appear that G 9/91 be not applicable (at least I like to think it this way :-)) to A. 100(b) I (still) cannot change my previous opinion that Opponent's appeal cannot be considered by the Board and will be rejected as inadmissible.

      @ Roel, is there any chance at this point that you may consider the possibility of revising the proposed solution taking into account the arguments punt forward above (also the reasoning set forth in item 4) and 5) of my post of 5 March 2018:-))?


      @ TK I like to think that [3] only state one possibility: ONLY claim 1 is attacked: the text goes: "The opponent attacked the patent under Article 1OO(b) EPC, demonstrating that ....fruit A and NO OTHER.."

      The demonstrating here states, in my view, how the patent was attacked: it was attacked "demonstrating that...A and no other".

      Therefore, here I do not see any possibility of an attack the whole patent, but just the intention of providing candidates with the information as to what and how was attacked.

      Matteo

      Delete
    21. @Anonymous 7 March 2018 at 19:53:
      you wrote "GL are not a legal text or legally binding strictu sensu".
      You seem to ignore Rule 22(1)(m) IPREE, which explicitly says that the Guidelines are part of the EQE syllabus.

      Delete
    22. @"Anonymous 7 March 2018 at 20:14 (Matteo)"

      No, I do understand your points, but they did not convince me, so I will not change our answer. The Guidelines reference strongly supports the view that the "extends to dependent claims (if...)" concept also holds for EPC requirements other than patentability in the strict sense (Art.52-57). The Guideline drafters select their wording carefully, as they are general instructions, and a user can to a large degree count on the Guidelines being correct. Also, I donot see any info in G 9/91 from which is can be seen that the EBoA intended to limit their conclusion to patentability in the struct sense, let alone that they meant to exclude Art.100(b) and Art.100(c) from the principle.

      I propose we now agree to disagree. I am not 100% certain of the answer, as -in whatever direction I go- I seem to be missing information from the paper (what was the extent? what were the first instance submissions? would sufficiency of claim 2 (...comprising A + B) be in prima facie doubt in view of the insufficiency of A only)? if so, why was it nevertheless allowed in first instance). In view of these doubts (which we discussed within our team, and we didnot always go in the same direction), we drafted the answer as we did and as I motivated further.

      So, unfortunately we cannot converge to an answer which all of the ones that contributed to this discussion. We will need to be patient and wait for the Examiner's Report to see what the Exam Committee's possibly solution is.

      Delete
  14. >G 9/91 refers to "patentability", not specifically to "Art.100(a)". I did not check in the earlier decision whether such narrow interpretation was meant, but I cannot derive that from G 9/912 itself. I see no reason why it would not apply to all Art.100 grounds.

    I am inclined to understand patentability as A. 100 (a) in view of the argument of the opponent which in the case at issue only opposed on the basis of A. 100 (a).

    Again for completeness a copy and paste from that decision:

    Summary of Facts and Submissions
    I. European patent No. 76 691, concerning certain anhydride polymers and imide polymers and processes for preparing them, was granted to Rohm and Haas Company in 1986. Within the period of nine months laid down in Article 99(1) EPC, notice of opposition to the patent was filed by BASF AG. In the statement pursuant to Rule 55(c) EPC the opponent requested that the patent be revoked to the extent it concerned polymers containing anhydride units on the ground that this subject-matter was not patentable within the terms of Articles 52 - 57 EPC (i.e. the ground referred to in Article 100(a) EPC).


    However, the referral from the president of the EPO was general i suppose. Therefore, I was in doubt myself. What made me take the decision was the presence of the further tests conducted both by patentee and opponent which would indicated fresh grounds and lack of prima facie.


    Thanks again,
    Matteo

    ReplyDelete
  15. Wouldn`t it make sense to withdraw the (inadmissable) appeal to get back 50% of the Appeal fees?

    ReplyDelete
  16. EP-BB2 claims a method of preparing a FFP by frying a ball of dough at a temperature in the range of 150-220°C, while PCT-FK1 discloses and claims a method of preparing a FFP containing fruit D by frying dough at between 155-250°C. If the wording "by frying a ball of dough at a temperature in the range of 150-220°C" would belong to the method step, then the method step should be different from PCT-FK1 as they merely discloses dough not a specific form of the dough. Therefore, the method according to EP-BB2 should be novel in light of PCT-FK1, but it is not inventive since the form of the ball is not related to a technical effect. Also the addition of the temperature range of 175-200°C would not result in an inventive step because a further (improved) technical effect (e.g. further lowering of the level of the acrylamide) is not be mentioned.

    ReplyDelete
  17. Hi Roel, could you please comment on the Last question from anonymous (3 March at 13:48). Thanks

    ReplyDelete
    Replies
    1. There were no questions in that post... but if you want me to comment on the post, yes, I can:

      The lack of mention of a new or enhanced effect of the sub-range (or the presence of an effect directly related to the effects mentioned in the application), would prevent the sub-range not to be novel as it fails the "purposive selection" criterion, and also not inventive.
      We have included both scenarios in our answer: an effect/advantage is in it, or it is not.

      Delete
    2. Hi Roel,

      thanks for the answer, and sorry I have forgotten the question mark. In the exam, I recognized immediately that the value in the prior art lies within the temperature range but I was struggling with the specific form of the dough, and I thought that this was part of the method step and therefore more specific than dough in general. So why do we not consider the form of the dough?

      Delete
    3. [012] also already discloses balls of dough in the first independent, apparatus, claim arranged to deposit valls of dough into a bath of oil. The second independent, method, claim does indeed not mention "ball of dough", but just "dough". Would have been better if that included also the words "a ball of", as kin all the other places in the paper where (balls of) douigh are fried... But, let's not be overly formalistic: the method of frying is clearly operated on the balls of dough that are deposited by the apparatus in the oil after the apparatus made the balls of dough. So the method comprising frying a ball of dough is (at least implicitly) disclosed as the method being executed with the apparatus.

      Roel, do you agree?

      Delete
    4. @Anonymous 5 March 2018 at 16:42:
      yes, I agree.

      Further, as long as the shape of the ball is not described (e.g. as being a sphere), a ball is the same/ cannot be distringuished from almost any shape, e.g. a clump, especially not in the context of "oil balls" (only very few of them are shapped as a sphere).

      Delete
  18. s there is quite some discussion on whether the decription of the contents of an application in the DII should always have the effects of the claims mentioned explicitly as being part of the application, I checked the DIIs exams of 2013-2017 for was done in the past:

    the DII 2013 (frying pans) was designed most nicely: everytime an application was introduced, the paper indicated explicitly for such application what was described, claimed and the effects or advantages of the newly added feature.

    in the DII 2014 (cream compositions), for half of the applications, the paper indicated explicitly that the effects were included in the application. For the other half, the effects were indicated elsewhere in the same paragraph or in the preceding paragraph.

    In the DII 2015 (printing machines), DII 2016 (coffee cups) and DII 2017 (metal beams), the effects were usually in the same or the precedings paragraph, and only occasionally explicitly indicated as part of an application.

    In all applications in the DII 2013 - 2017, the effects of different features were different effects or stronger versions of earlier effects. The effects were always needed to support (of attack) inventive step. Purposive selection was not tested in these DII papers.

    ReplyDelete
    Replies
    1. But do we have a purposive selection? The form of the dough is not disclosed in the prio art, so the method should be novel without regarding the temperature range.

      Delete
  19. PCT-FK1 showed method of preparing FPP containing D by frying dough at between 155 and 250 degrees C. The effect of the chosen fruit and the temperature range in combination is that products are crispy on the outside and flyffy on the inside for longer.
    EP-BB2 claims a method of preparing a FPP by frying a ball of dough at a temperature in a range of 150 to 220 degrees C. The description says that the roeferred range is in between 175 C and 200 C. We seem to be required to assume that the effects of [006] are also included in EP-BB2: acrylamide levels remain lower tgan 225 ppb regardless of the fruit content and type if temperature is lower than 220 degrees C; and if fruit C is present, any fryuing temperature within the usual range may be chosen. It is not at all clear whether "the usual range" is the claied range of 110-220 degrees C, or something else: that is simple not indicated anywhere in the paper.

    So, the 155 degrees end point destroys the novelty of the claimed range 150-220 degrees.

    But what to do with the preferred range 175-200?
    It is narrower than the prior art range 155-250 and "far removed" (although we xcannot be certain: the Guidelines say that what "narrow" and "far removed" is depends on the circumstances of the case, but there is nothing that can help us with that apart from the numerical values as such and the magic 220-point).
    But... the effect that is has is not specific for the range 175-200, but all over the range "below 220" [006]. There is no indication in [006] that the effect gets much higher in the preferred range 175-200 compared to 150-220.

    If/as the effect is not specific for the claimed range, but also occurs already over a part of the prior art range, can be then use this effect (from [006]) to argue purposive selection? Wouldnot that be strange? Does anyone know any case law on this (that is not so exotic, but that could realistically be considered part of the EQE syllabus)?

    ReplyDelete
    Replies
    1. GL (Nov 2016) G-VI, 8(ii) cites two decisions for the 3-point test for novelty of sub-ranges: T 198/84 and T 279/89, both relating to amending to sub-ranges w.r.t. prior art range in process claims.

      T 198/84, r.7. It remains to be established whether this view of novelty really entails more than just a formal delimitation of the process concerned vis-à-vis the state of the art. It would be delimited only in respect of the wording of the definition of the invention, but not in respect of its content, if the selection were arbitrary, i.e. if the selected range only had the same properties and capabilities as the whole range, so that what had been selected was only an arbitrary specimen from the prior art. This is not the case, since the effect of the substantial improvement in yield may be believed to occur __ONLY WITHIN THE SELECTED RANGE, BUT NOT OVER THE WHOLE KNOWN RANGE ("PURPOSIVE SELECTION")__. To prevent misunderstanding, it should be expressly emphasised that when examining so-called selection inventions as to novelty the Board adheres to the principle that the sub-range singled out of a larger range is new not by virtue of a newly discovered effect occurring within it, but must be new per se (cf. T 12/81 "Diastereomers/BAYER", OJ of the EPO 8/1982, 296, 303). An effect of this kind is not therefore a prerequisite for novelty; in view of the technical disparity, however, it permits the inference that what is involved is not an arbitrarily chosen specimen from the prior art, i.e. not a mere embodiment of the prior description, but another invention (purposive selection).

      T 279/89, r 4.1.3 Neither technical evidence demonstrating that the use of the 2,4'-isomer according to the claimed amounts leads to any particular properties, nor even a single argument in favour of a purposive selection have been provided by the Appellant. THE REFERENCE IN GENERAL TERMS IN THE DESCRIPTION OF THE APPLICATION IN SUIT to enhanced green strength properties cannot be regarded as evidence THAT THIS SPECIFIC IMPROVEMENT OCCURS ONLY FOR COMPOSITIONS CONTAINING AMOUNTS OF THE 2,4'-ISOMER WITHIN THE RANGES PRESENTLY CLAIMED. In this respect, the comparison between the properties achieved with the composition according to Example 4 and any of the compositions according to Examples 1 to 3 is not conclusive, since the latter are comparative examples, wherein the polyether used, according to the definition of THANOL SF-5505 at the bottom of page 7, contains hydroxyl end groups; this means that the advantages of compositions based on polyethers with amino end groups, such as JEFFAMINE T-5005, over compositions based on standard polyether polyols, referred to in Example 4 (page 9, lines 20 to 24) cannot be related to the amount of 2,4'-isomer in the MDI mixture, which is the critical compositional feature, and thereby demonstrate any difference with regard to the prior art teaching.

      Delete
  20. I may be incredibly off the mark here but...

    I noted that EP-BB2 discloses a mixer + nozzle. A screw conveyor is required for all nozzle shapes except a trumpet.

    PCT-BB2 discloses a broad mixer + any shaped nozzle but does not include that a screw conveyor is required. Thus, the priority is invalid for any nozzle shape in PCT-BB2, except for the specific (non-disclosed) trumpet nozzle + mixer because a screw conveyor is not required for this embodiment.

    Thus, a trumpet nozzle embodiment would be entitled to partial priority in PCT-BB2. But as there is no basis to amend to this subject matter it doesn't help in any way.

    EP-BB3 still needs a priority claim adding to the trumpet of EP-BB2

    JJ

    ReplyDelete
    Replies
    1. Hi JJ,

      [009] indicates "Consequently, we left out the description of the shape of the nozzle in PCT-BB2 in order to file a separate patent application directed to this invention. We added to the description that it is crucial that the nozzle is combined with a screw conveyor to expel the dough from the mixer." The crucial-comment has the result that also for the trumpet, PCT-BB2 requires to have the screw conveyor in the claim - the description says it is crucial = essential. The skilled person can only directly and unambigously derive that from PCT-BB2. PCT-BB2 does not indicate that the screw conveyor is not needed with a trumpet.
      So no trumpet + mixer w/o screw conveyor in PCT-BB2, so cannot clam prio from it.

      Note that EP-Bb2 is the first application for mixer + trumplet, so also for that reason, PCT-Bb2 can never be - the Art.87(4) exception does not help (see our answer and the discussion elsewhere on this blog).

      Delete
    2. Hi Roel,

      Thanks for your reply, I couldnt remember which in which of EP/PCT-BB2 it was disclosed that the screw conveyor was essential.

      I was expecting G1/15 to pop up somewhere so that's probably why i decided to put it in my answer. With hindsight it would have been wiser to leave it out as I wasn't totally sure. Fortunately it didn't really change my answer because there was nothing that could be done with the trumpet disclosure anyway.

      JJ

      Delete
    3. I think it was just a bit too early for G 1/15 to be part of the exam design (although it may have an effect of the answer - see C with G 1/15 and T 282/12 where it had a partial first application problem -- see our C blog): G 1/15 was only decided 29.11.2016, made available online on 02.02.2017 in the language of proceedings (English), and published in the OJ EPO only on 29.09.2017, so only about 1 month before the legal cutoff date of 31.10.2017. It is not so clear from the EQE syllabus when a G-decision is part of the syllabus, but I assume it is from the date of publication in the OJ EPO, as only then it is available in all three official EPO-languages.

      I would expect G 1/15 to be in next year's Pre-Exam, C and D though...

      Delete
  21. >Roel van Woudenberg8 March 2018 at 12:28
    @Anonymous 7 March 2018 at 19:53:
    you wrote "GL are not a legal text or legally binding strictu sensu".
    You seem to ignore Rule 22(1)(m) IPREE, which explicitly says that the Guidelines are part of the EQE syllabus.

    The rule says that " Candidates are expected to be familiar".
    It does not say that a non legal basis on the list becomes one.

    ReplyDelete
  22. What about:
    1. Claim Apparatus with trumpet nozzle + conveyor in EP Phase of PCT-BB2
    2. Claim Apparatus with trumpet DISCLAIMING the conveyor in the EP Phase of PCT-BB3 (without reviving EP-BB3) or, alternatively, in EP-BB3 (then reviving EP-BB3; with the disclaimer being "undisclosed") thereby being novel over PCT-BB2 Art 54(3).

    In the end this should provide full protection for the apparatus with trumpet nozzle with or without conveyor.

    Additional question: if EP-BB3 was revived. Would a claim in PCT-BB3 (in the EP Phase) directed to an apparatus with trumpet nozzel DISCLAIMING the conveyor be novel over the general disclosure in EP-BB3 of an apparatus with trumpet nozzle (leaving it open whether there is a conveyor or not)? I would say so...

    ReplyDelete
    Replies
    1. There are strict rules as to when "undislosed disclaimers" are allowed. Did you check whether those are fulfilled? Please share your argumentation.

      Novel maybe, but why inventive? Please share your argumentation.

      Delete
    2. PCT-BB2 Art is Art 54(3) EPC
      > undisclosed disclaimer is allowed
      > PCT-BB2 is only relevant to novelty not inventive step

      Delete
  23. In case anybody is reading this - the results are out!

    http://documents.epo.org/projects/babylon/eponot.nsf/0/7A1B9ED734DAFC4EC12582BB004674DC/$File/2018_RESULTS_MainEx_2018_Online_Publication.pdf

    ReplyDelete
  24. See also http://eqe-deltapatents.blogspot.com/2018/06/the-results-for-eqe2018-are-in.html and http://eqe-d.blogspot.com/2018/06/d-2018-results-are-in.html

    ReplyDelete
  25. The Examiner's Report is out!

    Had no chance to check ik out in detail, but a few things caught my eyes when glacing it through:

    - is shows what discussion was expected discussion on the appeal (inadmissible as not sufficiently substantiated; party as of right & prohibition of reformatio in peius; expected to amend to 1+2+3 with the test results proof of enablement -- there was no discussion in the ExRep as to the extent of opposition and whether the dependent claim were in it or not)

    - the references to the RPBA in the Possible Solution (also refer to the report on the Tutor meeting in epi Informatiom 4/2017)

    - The purposive selection criteria for novelty of sub-ranges was also extensively addressed in the discussions on the blog. The Possible Solution says: "PCT-BB2: To restore novelty over the disclosure in PCT-FK1, the claim should be limited to the temperature range 175-220°C. [...] The new range is narrow compared to the range of 155°C to 250°C disclosed by PCT-FK1 and sufficiently far removed from the end points of the known range. It is also a purposive selection, because the acrylamide level remains below 225 ppb if the claimed temperature range is used."
    The paragragh on the amended claim continues with an inventive step analysis and concludes with "The subjectmatter of the amended claims will thus in all likelihood be considered inventive." The "in all likelihood" is quite unusual in a Possible Soltion.

    - the Examiner's Report starts with a new paragraph: "Purpose and extent of the examiners’ report - The purpose of the present examiners’ report is to enable candidates to prepare for future examinations (cf. Article 6(6) of the Regulation on the European qualifying examination for professional representatives)."

    ReplyDelete
  26. Examiner's Report: “The filing fee and search fee for EP-BB3 were due by 26 February 2018”

    This is wrong, isn't it?

    Under EPC, “due” is the FIRST day on which they can validly be paid, not the LAST day! – see GL (2016/2017) A-X, 5.1.1

    ReplyDelete
    Replies
    1. The Guidelines does not define the term "is due".

      The Guidelines do define the terms "date date" and "payment in due time":

      GL (2017) A-X, 5.1.1 defines the word "due date":
      "In the EPC, the term "due date" has a special meaning, namely the first day on which payment of a fee may be validly effected, not the last day of a period for such payment (see A‑X, 6 "Payment in due time"). The due date for fees is generally laid down by provisions of the EPC or of the PCT. If no due date is specified, the fee is due on the date of receipt of the request for the service incurring the fee concerned.
      A fee may not be validly paid before the due date. The only exceptions to that principle are: [...]"

      The term "in due time" is defined in GL (2017_ A-X, 6.1:
      "A fee is considered to have been paid in due time if the date of payment (see A‑X, 4) fell on or before the last day of the relevant time limit"

      Delete
  27. Examiner's Report:

    "The time period for response to the Opponent’s grounds of appeal has not yet expired. BB should file a response within the time period of four months to be set by the Board."

    How can we know it would be 4 months? R.100(2) just says "period to be specified" and R.132(2) says just "at least two months", not four?

    ReplyDelete
    Replies

    1. It is in the Rules of Procedure of the Boards of Appeal - OJ 2007, 3536:
      Art.12(1) RPBA Appeal proceedings shall be based on:
      (a) the notice of appeal and statement of grounds of appeal filed pursuant to Article 108 EPC;
      (b) in cases where there is more than one party, any written reply of the other party or parties to be filed within four months of notification of the grounds of appeal;
      (c) any communication sent by the Board and any answer thereto filed pursuant to directions of the Board

      Delete
  28. Why design a whole DII on case law that is challenged by several decisions:

    the purposive selection test should not be part of the novelty test according to T 230/07, T 1130/09, T 40/11 and possibly more...

    See Case Law Book I.C.6.1, page 130-131 of English version of 2016 edition.

    Isn't it strange to built an exam on an old decision, T 198/84, that is under repeated recent scrutiny?

    I actually thought we were expected to use only the first 2 tests of Y 198/84. In other wordsm that we had to follow the recent decisions. Reason: the exam paper did not indicate that the PCT-BB2 included any effects. So when using th facts in tha paper and sticking to that -as required- it was not possible to argue purposive selection whereas it was possible to argue narrow and far removed...

    What do you guys think?

    V

    ReplyDelete
    Replies
    1. Both the Case Law Book as well as the Guidelines are explicitly listed in the EQE syllabus: the Case law Book in Rule 2 IPREE and the Guidelines in Rule 22(1) IPREE.

      So an answer based on either of them should attract full marks.

      Agree?

      Delete
  29. PCT-BB2: second non-medical use claim?

    Effect is low acrylamide irrespective of fruit mixture occurs over temperatures of 220 or lower (described), and claimed in range 150-220.

    Would a use claim including achieving the effect as a techical feature be novel - following G 2/88 "friction reducing oil"? Can that then also keep the whole range? I.e., is to possible to amend the claim from

    a method of preparing a fried food product (FFP) by frying a ball of dough at a temperature in the range of 150-220°C

    to

    use of a temperature in the range of 150-220°C when frying a ball of dough to prepare a fried food product (FFP) with an acrylamide level remaining lower than 225 ppb

    or even, as it is achieved at “220 or lower” in [006] as:

    use of a temperature below 220°C when frying a ball of dough to prepare a fried food product (FFP) with an acrylamide level remaining lower than 225 ppb

    ReplyDelete

Post a Comment